Which one of the following could be the meeting schedule for mornings and afternoons, in order from Wednesday through...

lsatprepper1 on June 8, 2020

Setup

I am struggling with the setup for this problem. Could someone explain the setup?

Replies
Create a free account to read and take part in forum discussions.

Already have an account? log in

Victoria on June 18, 2020

Hi @EliWhite8,

Happy to help!

We know that on W, T, and F, an accountant will hold exactly 5 meetings with 5 different clients - G, H, J, K, or L.

There are 6 available slots for meetings: one each morning and one each afternoon.

M: _ _ _
A: _ _ _
W T F

Now that we have the setup, let's go through the conditions.

Rule 1 - H and L must be scheduled for morning meetings.

Rule 2 - J must be scheduled the day before G
This means that J must have a meeting on W or T with G having a meeting the following day i.e. T or F, respectively.

Rule 3 - At most one other client can be scheduled between the meetings with G and K.
This means that G and K must either be scheduled consecutively, on consecutive mornings, or on consecutive afternoons.

Now let's go through the answer choices.

We can immediately eliminate answer choices (B) and (C) because H and L must be scheduled for morning meetings.

We can also eliminate answer choice (A) because J must be scheduled for the day before G not the day after.

Finally, we can eliminate answer choice (D) because there are two clients (J and H) scheduled between K and G. Rule 3 tells us that a maximum of one client may be scheduled between the two.

Therefore, the correct answer is (E).

Hope this is helpful! Please let us know if you have any further questions.

Gabe85 on September 11, 2020

Are there any deductions that can be made? I found myself testing multiple possibilities for multiple scenarios on every question and it was way too time consuming to finish with a time limit.

Seo on September 29, 2020

Same as above. This game took me way longer than I expected, because I was testing each scenarios as I go.
What kind of deductions can be made here?